LSAT and Law School Admissions Forum

Get expert LSAT preparation and law school admissions advice from PowerScore Test Preparation.

 Administrator
PowerScore Staff
  • PowerScore Staff
  • Posts: 8915
  • Joined: Feb 02, 2011
|
#22890
Complete Question Explanation

Strengthen. The correct answer choice is (B)

The initial premise is that when a business has linked computer systems with identical operating systems, a vandal who accesses one computer has access to the data on all of the company's computers.

The vandal can then introduce a "virus" to destroy data.

The stimulus then establishes that minor variations in operating systems software could virtually eliminate the vandal's ability to access the computers all at once, and that the computers would still be compatible enough for business purposes.

The argument concludes that businesses should implement minor variations in the operating system software of its various computers.

You should notice that the stimulus has already covered the most likely commonsense objection to varying operating systems by pointing out that compatibility is not an issue. However, it is still entirely possible that there are concerns other than compatibility that could affect the argument. There could be other solutions to the problem, or the proposed solution might be overly costly, or inefficient in some way. Since you are asked to strengthen the argument, it is likely that you will simply choose an answer that eliminates a concern, which is the Defender model for assumption and strengthen answers.

Answer choice (A): This choice seeks to establish that it is easy for a vandal to gain access across businesses. However, the stimulus concerned access within a single business, so this choice is incorrect. If you thought that this choice made it more likely that businesses were at risk without varying their operating software, you should realize that answer choice (A) only refers to an increase in compatibility, and does not infer that the systems are similar enough for a multi-business attack. Furthermore, the stimulus was concerned with how to deal with the existing risk. Proving that the risk is even greater does not strengthen the idea that the solution proposed in the stimulus is correct.

Answer choice (B): This is the correct answer choice. If it were true that it would be cheaper to simply repair the damage from attacks than to replace our operating systems, it might not make sense to replace our systems. Since this choice points out that repairs following an attack would be more expensive than preventing the attack, answer choice (B) eliminates the concern that the preventative measures might be more costly than simply enduring the problem.

Answer choice (C): This choice is entirely irrelevant. The LSAT test writers have included this choice on the possibility that you missed the premise that eliminated compatibility as an issue. If you chose this answer, you probably need to be more careful when reading the stimulus.

Answer choice (D): This choice could actually suggest that there is no way to defend against attacks in general, thereby challenging the conclusion. You should eliminate this choice, as it either is irrelevant or weakens the argument.

Answer choice (E): This choice is irrelevant. Just as in answer choice (C), this response is for those who fell prey to a careless reading, and neglected that compatibility is a non-issue.
 Blueballoon5%
  • Posts: 156
  • Joined: Jul 13, 2015
|
#19386
18. Computer operating system...

My Question: I do not understand why choice A and C are wrong. I do not understand why choice B is correct.

For answer choice A, the answer key explains that this is out of scope. However, I thought that we can bring in new information in strengthen questions? For example, if choice A was true, than it would be more advisable to implement such variations to avoid viruses spreading though multiple businesses.

For answer choice C, I do not see how this is different from answer choice B.

For answer choice B, I do not understand why we can bring in the new information of "cost," but we are unable to bring in the new information of "multiple businesses" in choice A.

Thanks!!! :-D
 Jon Denning
PowerScore Staff
  • PowerScore Staff
  • Posts: 904
  • Joined: Apr 11, 2011
|
#19405
Hey blue,

Thanks for the question! I'll hopefully be able to clarify those answer choices for you here.

This is a Strengthen question, so we're looking for an answer choice that supports the author's conclusion. So let's start with the conclusion and see what's being argued here.

The author believes it's advisable for businesses to implement minor variations in their operating system software. Why? This would make unauthorized access to all the company's computers virtually impossible and thus presumably prevent hacking, all while maintaining computer compatibility for the business. So we need an answer choice that helps this idea in some way.

Let me start with the correct answer, B, and explains how it helps, and then look at the two wrong answers you mention and see how to eliminate them. Answer choice B strengthens by countering a potential objection, namely "the cost/difficulty in recovering from a hack or virus is WAY less than the cost/difficulty of implementing minor software variations to prevent a hack." If that was true then this argument is in real trouble, as the prevention would be more harmful (expensive in this case) than the thing you're trying to prevent (the hack)! So B ensures that's not the case by saying that preventing the problem (i.e. implementing software changes) is less expensive than correcting the problem (fixing the invasive hack), and therefore lends credibility to the author's suggestion. In short, prevention is cheaper than repair, so the author's suggestion to prevent seems reasonable.

Answer choice A, on the other hand, talks about compatibility of software between businesses, something that is entirely irrelevant to this argument, which is only about compatibility within a single business. What it might do outside of that business simply has no effect on this argument. It's fine to bring in new information—C talks about expense, for instance, and is correct—but it's not okay if that information doesn't apply to the argument itself, and A's information doesn't (hence the "out of scope" explanation).

Answer choice C also fails to address the argument made here: stating that it's not costly for a business to maintain incompatible operating systems is irrelevant, since this argument is about software that is, and still will be, compatible. That's the point in the second-to-last sentence where it says variations can be created without losing any compatibility. C, like A, addresses facts/scenarios that aren't a part of the argument, and thus are immediately wrong.

I hope that helps!
 LSATer
  • Posts: 47
  • Joined: Nov 13, 2016
|
#33919
How do you distinguish this as a strengthen question and not a must be true? This question stems tripped me up.
 Kristina Moen
PowerScore Staff
  • PowerScore Staff
  • Posts: 230
  • Joined: Nov 17, 2016
|
#33927
Hi LSATer,

Great question. It's very important distinguish between a Must Be True and a Strengthen, and they can sound very similar!

Which one of the following, if true, supports the conclusion in the passage?
vs.
Which one of the following, if true, is supported by the passage above?

The first stem is a Strengthen. The answer choices are doing something to the stimulus. They are supporting the stimulus. In this question type, you will be looking for new information that will strengthen the conclusion. That is what we see on this question.

The second stem is a Must Be True. The stimulus is doing something to the answer choices. The answer choices are being supported by the stimulus. In this case, you are not looking for new information, but for something that you can infer from the stimulus.
 LSATer
  • Posts: 47
  • Joined: Nov 13, 2016
|
#33949
Yeah, this is very tricky. :) Thank you for your help.
 yrivers
  • Posts: 68
  • Joined: Mar 15, 2017
|
#34455
Could you expand on why E in wrong? Specifically, I am confused because the stimulus states that "it is advisable for businesses to implement such VARIATIONS."

I read that to mean that computer systems need variations (as opposed to shared systems with no variation where virus/vandal users are more likely to take advantage). So if businesses don't need to share data among their internal computer systems (E) and make it easier/support the idea of pursuing variations. If you're on the same system, aren't you sharing systems/data?

Am I thinking of "sharing data" interchangeably with no variation in system, when I shouldn't be?
 Emily Haney-Caron
PowerScore Staff
  • PowerScore Staff
  • Posts: 577
  • Joined: Jan 12, 2012
|
#34505
Hi yrivers,

Thanks for the question! The trick to E is that we have no basis for thinking that sharing data among systems is hindered in any way by implementing variations in operating system software. Take a look at the stimulus again - you'll notice that E doesn't actually impact the argument at all, because even if all businesses very much need to share data, we don't have a reason to think they wouldn't be able to after variations are introduced. Hope that helps!

Get the most out of your LSAT Prep Plus subscription.

Analyze and track your performance with our Testing and Analytics Package.